LSAT and Law School Admissions Forum

Get expert LSAT preparation and law school admissions advice from PowerScore Test Preparation.

 dbrowning
  • Posts: 26
  • Joined: Jun 18, 2019
|
#65703
Hi, while working through the LRB, I came across one of the questions in the problem set that confused me. The second question (562 in 2019 version) is a parallel flaw, and the correct answer D seems to contain wording inconsistent with the stimulus. In the stimulus, the author claims that the country "will not improve". In the correct answer, the outside temperature "cannot rise". Since will not does not imply cannot, I do not see how the answer precisely parallels the stimulus.

Is D correct in this case because the flaw in the answer choice still matches the flaw in the stimulus?

In general, the distinction between cannot and will not is important, but is this distinction important to LSAT authors as it relates to parallel questions?
 Brook Miscoski
PowerScore Staff
  • PowerScore Staff
  • Posts: 418
  • Joined: Sep 13, 2018
|
#65722
dbrowning,

When doing a parallel question, avoid trying to equate the answer choice with the stimulus. You should focus on whether the reasoning is parallel rather than whether "will not" always equates with "cannot." What you're looking for is how each word relates to the logic in its own argument. It is true that you should match the level of certainty of certain words--for example, you would care that "will not" is not similar to "might not."

You might also consider whether in the stimulus the context of "will not" is that the author is claiming it's impossible for the country to improve.

Get the most out of your LSAT Prep Plus subscription.

Analyze and track your performance with our Testing and Analytics Package.